1989 AIME Problems/Problem 10

Revision as of 23:11, 24 February 2007 by Minsoens (talk | contribs)

Problem

A sample of 121 integers is given, each between 1 and 1000 inclusive, with repetitions allowed. The sample has a unique mode (most frequent value). Let $D^{}_{}$ be the difference between the mode and the arithmetic mean of the sample. What is the largest possible value of $\lfloor D^{}_{}\rfloor$? (For real $x^{}_{}$, $\lfloor x^{}_{}\rfloor$ is the greatest integer less than or equal to $x^{}_{}$.)

Solution

This problem needs a solution. If you have a solution for it, please help us out by adding it.

See also